Quadratic Discriminant

Algebra Level 4

x 2 + 2 x 11 x 3 \frac { { x }^{ 2 }+2x-11 }{ x-3 }

For all real x x which of the following interval values can not be attained by the above expression?

( 3 , 1 ) (-3,1) ( 4 , 3 ) (-4,3) ( 2 , 3 ) (2,3) ( 4 , 12 ) (4,12)

This section requires Javascript.
You are seeing this because something didn't load right. We suggest you, (a) try refreshing the page, (b) enabling javascript if it is disabled on your browser and, finally, (c) loading the non-javascript version of this page . We're sorry about the hassle.

2 solutions

Consider x 2 + 2 x 11 x 3 = y \frac { { x }^{ 2 }+2x-11 }{ x-3 } =y Then we have, x 2 + 2 x 11 = x y 3 y x 2 + ( 2 y ) x 11 + 3 y = 0.......... ( i ) \quad { x }^{ 2 }+2x-11=xy-3y\\ \Rightarrow { x }^{ 2 }+(2-y)x-11+3y=0..........(i) Now, since we have to find the range of y y where x x is real, we consider the discriminant ( 2 y ) 2 4.1. ( 3 y 11 ) {{ (2-y) }^{ 2 }-4.1.(3y-11)} of equation (i) and observe that it should be greater than or equal to zero since equation (i) has to give real solutions for x x . Thus ( 2 y ) 2 4.1. ( 3 y 11 ) 0 y 2 16 y + 48 0 ( y 4 ) ( y 12 ) 0 y 4 , o r , y 12 y ( 4 , 12 ) { (2-y) }^{ 2 }-4.1.(3y-11)\ge 0\\ \Rightarrow { y }^{ 2 }-16y+48\ge 0\\ \Rightarrow (y-4)(y-12)\ge 0\\ \Rightarrow y\le 4,\quad or,\quad y\ge 12\\ \Rightarrow y\notin (4,12) Hence the result.

Pulkit Gupta
Dec 14, 2015

We first rewrite the numerator as x 2 + 2 ( x 3 ) 5 x 3 \LARGE\frac{x^{2} + 2( x -3) -5}{x-3} = 2 \LARGE 2 + x 2 5 x 3 \LARGE \frac{x^{2} -5}{x-3}

Let x 2 5 x 3 \LARGE \frac{x^{2} -5}{x-3} = y \LARGE y

Now proceed along the lines of what Kuldeep has done. Just remember that 2 must be added to lower and upper limits obtained from this expression.

0 pending reports

×

Problem Loading...

Note Loading...

Set Loading...